Double sums and convergence to $infty$.












1












$begingroup$


I am having trouble proving/disproving the following:



Let ${f_n}_{n=1}^infty$ be a sequence with $f_n:mathbb{N} rightarrow mathbb{R}^+$.
Suppose $sum_{n = 1}^infty big (sum_{k=1}^infty f_n(k)) = infty$. Prove or disprove that $sum_{k = 1}^infty big (sum_{n=1}^infty f_n(k)) = infty$.



I suspect it is true, and have tried using the definition of a series diverging to $+ infty$, but I don't know how to handle the inner sum. Any help would be appreciated.










share|cite|improve this question









$endgroup$








  • 1




    $begingroup$
    Try proving the contrapositive (that is, if one is finite, then they’re both finite [and have the same value]). Hint: all of the terms are positive.
    $endgroup$
    – Clayton
    Jan 21 at 0:44










  • $begingroup$
    With the contrapositive, I don't have to worry about divergence (or oscillations) since all the terms are positive , right?
    $endgroup$
    – user439126
    Jan 21 at 0:49










  • $begingroup$
    Correct. ${}{}$
    $endgroup$
    – Clayton
    Jan 21 at 0:55










  • $begingroup$
    Great, thanks a bunch.
    $endgroup$
    – user439126
    Jan 21 at 0:59
















1












$begingroup$


I am having trouble proving/disproving the following:



Let ${f_n}_{n=1}^infty$ be a sequence with $f_n:mathbb{N} rightarrow mathbb{R}^+$.
Suppose $sum_{n = 1}^infty big (sum_{k=1}^infty f_n(k)) = infty$. Prove or disprove that $sum_{k = 1}^infty big (sum_{n=1}^infty f_n(k)) = infty$.



I suspect it is true, and have tried using the definition of a series diverging to $+ infty$, but I don't know how to handle the inner sum. Any help would be appreciated.










share|cite|improve this question









$endgroup$








  • 1




    $begingroup$
    Try proving the contrapositive (that is, if one is finite, then they’re both finite [and have the same value]). Hint: all of the terms are positive.
    $endgroup$
    – Clayton
    Jan 21 at 0:44










  • $begingroup$
    With the contrapositive, I don't have to worry about divergence (or oscillations) since all the terms are positive , right?
    $endgroup$
    – user439126
    Jan 21 at 0:49










  • $begingroup$
    Correct. ${}{}$
    $endgroup$
    – Clayton
    Jan 21 at 0:55










  • $begingroup$
    Great, thanks a bunch.
    $endgroup$
    – user439126
    Jan 21 at 0:59














1












1








1





$begingroup$


I am having trouble proving/disproving the following:



Let ${f_n}_{n=1}^infty$ be a sequence with $f_n:mathbb{N} rightarrow mathbb{R}^+$.
Suppose $sum_{n = 1}^infty big (sum_{k=1}^infty f_n(k)) = infty$. Prove or disprove that $sum_{k = 1}^infty big (sum_{n=1}^infty f_n(k)) = infty$.



I suspect it is true, and have tried using the definition of a series diverging to $+ infty$, but I don't know how to handle the inner sum. Any help would be appreciated.










share|cite|improve this question









$endgroup$




I am having trouble proving/disproving the following:



Let ${f_n}_{n=1}^infty$ be a sequence with $f_n:mathbb{N} rightarrow mathbb{R}^+$.
Suppose $sum_{n = 1}^infty big (sum_{k=1}^infty f_n(k)) = infty$. Prove or disprove that $sum_{k = 1}^infty big (sum_{n=1}^infty f_n(k)) = infty$.



I suspect it is true, and have tried using the definition of a series diverging to $+ infty$, but I don't know how to handle the inner sum. Any help would be appreciated.







real-analysis






share|cite|improve this question













share|cite|improve this question











share|cite|improve this question




share|cite|improve this question










asked Jan 21 at 0:37









user439126user439126

1196




1196








  • 1




    $begingroup$
    Try proving the contrapositive (that is, if one is finite, then they’re both finite [and have the same value]). Hint: all of the terms are positive.
    $endgroup$
    – Clayton
    Jan 21 at 0:44










  • $begingroup$
    With the contrapositive, I don't have to worry about divergence (or oscillations) since all the terms are positive , right?
    $endgroup$
    – user439126
    Jan 21 at 0:49










  • $begingroup$
    Correct. ${}{}$
    $endgroup$
    – Clayton
    Jan 21 at 0:55










  • $begingroup$
    Great, thanks a bunch.
    $endgroup$
    – user439126
    Jan 21 at 0:59














  • 1




    $begingroup$
    Try proving the contrapositive (that is, if one is finite, then they’re both finite [and have the same value]). Hint: all of the terms are positive.
    $endgroup$
    – Clayton
    Jan 21 at 0:44










  • $begingroup$
    With the contrapositive, I don't have to worry about divergence (or oscillations) since all the terms are positive , right?
    $endgroup$
    – user439126
    Jan 21 at 0:49










  • $begingroup$
    Correct. ${}{}$
    $endgroup$
    – Clayton
    Jan 21 at 0:55










  • $begingroup$
    Great, thanks a bunch.
    $endgroup$
    – user439126
    Jan 21 at 0:59








1




1




$begingroup$
Try proving the contrapositive (that is, if one is finite, then they’re both finite [and have the same value]). Hint: all of the terms are positive.
$endgroup$
– Clayton
Jan 21 at 0:44




$begingroup$
Try proving the contrapositive (that is, if one is finite, then they’re both finite [and have the same value]). Hint: all of the terms are positive.
$endgroup$
– Clayton
Jan 21 at 0:44












$begingroup$
With the contrapositive, I don't have to worry about divergence (or oscillations) since all the terms are positive , right?
$endgroup$
– user439126
Jan 21 at 0:49




$begingroup$
With the contrapositive, I don't have to worry about divergence (or oscillations) since all the terms are positive , right?
$endgroup$
– user439126
Jan 21 at 0:49












$begingroup$
Correct. ${}{}$
$endgroup$
– Clayton
Jan 21 at 0:55




$begingroup$
Correct. ${}{}$
$endgroup$
– Clayton
Jan 21 at 0:55












$begingroup$
Great, thanks a bunch.
$endgroup$
– user439126
Jan 21 at 0:59




$begingroup$
Great, thanks a bunch.
$endgroup$
– user439126
Jan 21 at 0:59










1 Answer
1






active

oldest

votes


















0












$begingroup$

Hint : Yes it is true, because the terms on the sum are positive. So you can show that both terms are equal to the supremum of the finite sums :
$$ sup_{K, N in mathbf {N}} sum_{n=0}^N sum_{k=0}^K f_n (k) $$






share|cite|improve this answer









$endgroup$













    Your Answer





    StackExchange.ifUsing("editor", function () {
    return StackExchange.using("mathjaxEditing", function () {
    StackExchange.MarkdownEditor.creationCallbacks.add(function (editor, postfix) {
    StackExchange.mathjaxEditing.prepareWmdForMathJax(editor, postfix, [["$", "$"], ["\\(","\\)"]]);
    });
    });
    }, "mathjax-editing");

    StackExchange.ready(function() {
    var channelOptions = {
    tags: "".split(" "),
    id: "69"
    };
    initTagRenderer("".split(" "), "".split(" "), channelOptions);

    StackExchange.using("externalEditor", function() {
    // Have to fire editor after snippets, if snippets enabled
    if (StackExchange.settings.snippets.snippetsEnabled) {
    StackExchange.using("snippets", function() {
    createEditor();
    });
    }
    else {
    createEditor();
    }
    });

    function createEditor() {
    StackExchange.prepareEditor({
    heartbeatType: 'answer',
    autoActivateHeartbeat: false,
    convertImagesToLinks: true,
    noModals: true,
    showLowRepImageUploadWarning: true,
    reputationToPostImages: 10,
    bindNavPrevention: true,
    postfix: "",
    imageUploader: {
    brandingHtml: "Powered by u003ca class="icon-imgur-white" href="https://imgur.com/"u003eu003c/au003e",
    contentPolicyHtml: "User contributions licensed under u003ca href="https://creativecommons.org/licenses/by-sa/3.0/"u003ecc by-sa 3.0 with attribution requiredu003c/au003e u003ca href="https://stackoverflow.com/legal/content-policy"u003e(content policy)u003c/au003e",
    allowUrls: true
    },
    noCode: true, onDemand: true,
    discardSelector: ".discard-answer"
    ,immediatelyShowMarkdownHelp:true
    });


    }
    });














    draft saved

    draft discarded


















    StackExchange.ready(
    function () {
    StackExchange.openid.initPostLogin('.new-post-login', 'https%3a%2f%2fmath.stackexchange.com%2fquestions%2f3081339%2fdouble-sums-and-convergence-to-infty%23new-answer', 'question_page');
    }
    );

    Post as a guest















    Required, but never shown

























    1 Answer
    1






    active

    oldest

    votes








    1 Answer
    1






    active

    oldest

    votes









    active

    oldest

    votes






    active

    oldest

    votes









    0












    $begingroup$

    Hint : Yes it is true, because the terms on the sum are positive. So you can show that both terms are equal to the supremum of the finite sums :
    $$ sup_{K, N in mathbf {N}} sum_{n=0}^N sum_{k=0}^K f_n (k) $$






    share|cite|improve this answer









    $endgroup$


















      0












      $begingroup$

      Hint : Yes it is true, because the terms on the sum are positive. So you can show that both terms are equal to the supremum of the finite sums :
      $$ sup_{K, N in mathbf {N}} sum_{n=0}^N sum_{k=0}^K f_n (k) $$






      share|cite|improve this answer









      $endgroup$
















        0












        0








        0





        $begingroup$

        Hint : Yes it is true, because the terms on the sum are positive. So you can show that both terms are equal to the supremum of the finite sums :
        $$ sup_{K, N in mathbf {N}} sum_{n=0}^N sum_{k=0}^K f_n (k) $$






        share|cite|improve this answer









        $endgroup$



        Hint : Yes it is true, because the terms on the sum are positive. So you can show that both terms are equal to the supremum of the finite sums :
        $$ sup_{K, N in mathbf {N}} sum_{n=0}^N sum_{k=0}^K f_n (k) $$







        share|cite|improve this answer












        share|cite|improve this answer



        share|cite|improve this answer










        answered Jan 21 at 0:48









        DLeMeurDLeMeur

        3148




        3148






























            draft saved

            draft discarded




















































            Thanks for contributing an answer to Mathematics Stack Exchange!


            • Please be sure to answer the question. Provide details and share your research!

            But avoid



            • Asking for help, clarification, or responding to other answers.

            • Making statements based on opinion; back them up with references or personal experience.


            Use MathJax to format equations. MathJax reference.


            To learn more, see our tips on writing great answers.




            draft saved


            draft discarded














            StackExchange.ready(
            function () {
            StackExchange.openid.initPostLogin('.new-post-login', 'https%3a%2f%2fmath.stackexchange.com%2fquestions%2f3081339%2fdouble-sums-and-convergence-to-infty%23new-answer', 'question_page');
            }
            );

            Post as a guest















            Required, but never shown





















































            Required, but never shown














            Required, but never shown












            Required, but never shown







            Required, but never shown

































            Required, but never shown














            Required, but never shown












            Required, but never shown







            Required, but never shown







            Popular posts from this blog

            Mario Kart Wii

            What does “Dominus providebit” mean?

            Antonio Litta Visconti Arese